You are on page 1of 33

IP

A
WP
HP* :
%

1e

^
XJ

pprORCED

BEA M REIN FOR CED


FOR TEN SION

229

CONCRETE DESIGN
As * p bd

( tension reinforcement index )

fc p bd fy
= 0(0.85) 0 85 fc
fy
AS
d - 0.85 fc' b( 2 )

F U

**

o V

Mu

fy
p
bd
.
r
P
' co bd
fc
0
=
Mu
[d - 2 fc- (Q.85) b

r
2 bd
fc
co
0
Mu =
[ 2
'

-O-O- -

p b2d 2 fy

(0.85) b

2
bd
co
' co bd fc
0
=
Mu
2 (0.85)

0 85/

AAu = 0 fc

'

C=0.85/ tai

2
co bd

( l - YJ )

Mu = o fc' bd 2 ( 1 - 0.59 co)


'
fc
Rn = ( 1 - 0.59 co)
Rn = coefficient of resistance

JW,

C = 0.85 fc' ab
(compressive force of concrete )
T = As fy

bd 2

Mu =

Rn

2
0 bd

'

Rn

( tensile force of steel reinforcem ;nt )


Hi

IFh = C
T=C
As fy = 0.85 fc' ab

[5

(steel ratio )

Mu * o As fy ( d - a / 2)

Asfy

0.85 fc' b

( ultimate moment capacity)

Mu = 0 C (d - a / 2 )
Mu = 0 0.85 fc' ab ( d - a / 2)
..

o 0.85 fc' As fy b
( d a / 2)
Mu "
SSfc

-V

'

Mn = Asfy (d - a / 2 )
( nominal moment capacity )

230

steel
and
A. Design for dimension
reinforcements.

Given data :
M,
Ultimate moment
Concrete strength
fe'
Yield strength of steel . . . . f

0 003

c6 =

"

0,003(200,000)
f + 0.003(200,000)
600 d

A,

fi + 600

N=
Is

..

Ste

ft
ft
P

Pt

8. h)v

<

fcr

_ 0.85 f/ R 600

(Ur
(Pi

f ( fy + 600)

(balanced steel ratio)


0 85 Vs 600
Pt -~ f (
, 600 + fy )

Wir
&

Assume p - 0.5 pb

OSSfc -

fie

*'

Shply supported
-

Co

pt

Step 2. Assume min. thickness of be

Sek

C=T
f,
0.85 f ab =
0- 85 f. R cb b = Af
A
p6 X
bd
0.85 f/ Bctb = p( bdf>
0.85
f
act
.
b
f, d
0.85 r 8 (600) d
Pb "
, f, + 600)

(0

ix

'

n,

CO

Stepl . Compute balanced steel ratio :


In a balanced section, steel starts yielding
when concrete just reaches its ultimate strain
capacity and commences to crush. At the
start of failure , the permissible extreme fiber
compressive strain is 0.003, while the tensile
strain in the steel equals the yield strain

n si

U*3

-L + 0.003

o 85fc -

yil

d
c
0.003 , + 0.003
0 003
c6
e + 0.003
0.003

Required :
b
a) Width of beam
b) Effective depth of beam . . . d
c) Steel reinforcement needed At

SI

REINFOR

,
**

T-

One end continuous

So<h end continuous

Cantilever beams

~z

16
L

fia
21

,
BT

231

rONCRETE DESIGN

eiNWBCI

m
62.5
covering
steel
o
U

0.003

0.85fc

h * d + 62.5
d * h 62.5

Compute for "b"u8ing :


(o (1 0.59(u)
bdJ
f
0
,
'
Mu

A, = pbd
Select no. of bars:
(

,
Sf
*

T=

Step 3. Steel reinforcement needed

(d a/ 2 )

a) Tension Reinforcement index

p=
F

d N

-bd

N = no. of bars
d = diameter of bars
Step 4. Check for pmt and p

= 075 P
*
>
Pim Pntn
1.4
P > P^ = T

B. Investigation of Beams Reinforced


for Tension if Steel will yield:
(Under Reinforced )
(pmln < p < pmax)
Given data :
Width of beam
Effective depth of beam . .
Steel reinforcement
Concrete strength
Yield strength of steel . . . .
Span of beam

l
^

f) Safe concentrated load

0.85 fc ' ab

c) Ultimate moment
a = lic
_a

e,

b
d

IJ

_ 0.003

c
d-c
When:
, > 0.005
Use 0 = 0.90

At
f'
f
L

* 0.7 + <e -0.002)25<V3

a) Tension reinforcement index , . w


Depth of compr ession block . a
c) Ultimate moment

Coefficient of resistance

Required :

b) Depth of compression block


Compare with p
^
p < pmax steel bars will yield
p > psteel bars will not yield
*
0 =1
0.85 fc' ab = A fy

Mu

R
P

Other

When:
, > 0.002

carolled

< 0.005

Use 0 = 0.65 + ( - 0.002)


When:
< 0.002
Use 0 = 0.65

0 a 0.65

( -0.002)250/3

coo

REINFORCED n

232

s) Balanced Steel ratio

Nominal Moment Capacity


)
2
d
/
3
(
A
=
,
fy
Mn
Ultimate moment capacity:
)
/
2
(
a
d
0
=
Ma At

L d M Ir C s a i*
B = 0.85 -

but not less than 0.65


_ 0.85 f ' B 600
Pb
600
)
U

d) Coefficient of resistance
2
)
1
bd
(
u
a
0
0.5
r
=
9
<
<
Mu
R

,
*

"

0 bd

'

A,

p < pmt steel bars will not yield

Use: f

/lr

b) Depth of compression block

e) Safe concentrated load

TrJ

'

005 ft' 30)

C. Investigation of Beams Reinforced


for Tension if Steel will not yield:
(Over Reinforced )
(pmln ^ Pmtx < p )
Given data :
Width of beam
b
Effective depth of beam . . . d
Steel reinforcement
As
Concrete strength
f'
Yield strength of steel . . . . f
Span of beam
L
Required :
a) Balanced Steel ratio .
b) Depth of compression block . .
a
c) Ultimate moment
y

SHi
ZX f

"

Dr

*
e

C= 85 /C'4

T=AsFy

_ 600 (d - c)

a) Depth

C=T
0.85 f ' ab = A f

a = Gc (d

c
Solve fore: (quadratic equation)
a = Bc

D Balanced Condition

Balanced steel area


Ultimate moment
for balanced condition

0.8

^^

'

mai

E*
P*

Given data :
Width of beam
b
Effective depth of beam . . . d
Steel reinforcement
A,
Concrete strength
f'
Yield strength of steel . . . . f
Span of beam
L

b)
c)

ks

c) Ultimate moment
Mu = 0 A f (d - a/2)

Required :
a) Depth of compression block

b) Balant
C =T
O Sf ' ab

1 5,

17

233

Tension Steel Yields

0.85 fc '

&
30) r

3. Problem:
(d -al 2 )

r
c

ssionbiOCk

fr

will not yield

/E , 0.003

a) Depth of compression block :

A rectangular concrete beam has a width of


250 mm and a total depth of 450 mm. It is
2
mm
5
187
reinforced with a total steel area of
placed at an effective depth of 375 mm.
fc' = 27.6 MPa, fy = 414.7 MPa.

Determine the depth of compression


block.
Determine the moment capacity reduction
factor.
(D Determine the safe live load that the beam
could carry in addition to a dead load of
20 kN/m if it has a span of 6 m.

80Od
f + 600

a = lie (depth of compression block )

)(d - c)
c
equation)

Solution:
Depth of compression block :

b) Balanced steel area

C= T
0.85 T ab = As6
As

As# =

X
A =1873

'
balanced steel area

Max. A - 0.75 Asb


J!tnate moment for balanced condition:
M = 0 A fy (d - a/2)

..

Check:
' li (600)
f
5
0.8
.
n
p

>

"

1=

Av

M.

> ' <

OSSVab

Assuming the steel yields:


T=C
A fy = 0.85 fc ab
1875(414.7) = 0.85(27 -6)(a) 250)
a = 132.58 mm

Condition:
b
..d
A,

/r

fy

c) Ultimate Moment for Balanced

..

T(MOH )

<

Moment capacity reduction factor.


a = UC
132.58 = 0.85 C
C = 155.98

(baiancedsteelrali0)

0.001

C =155.98!

bd

- -5
:i 9 o:
t/

75

VI

REINFORCED CONCRETE

234

0.003

91Q 09

IRSQft

e, =1

E
414.7

, = 200 000

= 0.0020735 < 0.0042125

Mu

8
2

6
(
)
Wu
200.2
8
W= 44.48 kN/m

Pm

Pm
p-

+
LL
1.7
DL
1.4
=
Wu

44.48 = 1.4(20) + 1.7 LL


LL = 9.69 kN / m

Steel yields:
0

_ WUL

E { = 0.0042125 > 0.002 but < 0.005

st

0.7 + ( -0.002 )250/3

0.90
c

i
i

0.70
0.65

Tension Steel
Does Not Yield

Spiral
Other

/
0 = 0.65 +
'

( 0.002) 250/3

Compression Transition

controlled
0.002

cV== 0.60

SI

Tension
controlled
e == 0.005
c/d 0.375

Since e is between 0.002 and 0.005,


tiiis value is within the transition range
between compression controlled section
and tension controlled section.

Use 0 = 0.65 + ( - 0.002)

250
0 = 0.65 + (0.0042125 - 0.002)
3
0 = 0.834

4. Problem :

A rectangular beam has a width of 280 mm


and an effective depth of 500 mm. It has a
reinforcement area of 4072 mm2 at the bottom.
Balanced steel ratio p t> = 0.026. fc' = 25 MPa,
fy = 414 MPa.

C=

0.8!

0.8!

Determine the depth of compression


block.
Determine the tensile force carried by the
steel bars.
Determine the resisting moment capacity
of the beam, checked for moment capaoty

505
2

c
c=
a=
a=
as

reduction factor.
i

Solution:
Depth of compression block.

Live load it could support:

0 S5fc

(
d
0
")
=
y
M
Mu
Mu = 0.834(1875)(414.7)(375 mm
N
x
.
106
200.2
=
Mu
m
.
kN
200.2
=
Mu

if .
,v

d=500

500
I

T
X
T,

r.
DESIGN

Resisting moment capacity of the


beam, checked for moment capacity
reduction factor.
Mu = 0 As fy (d - a/2)
Mu = 0 T (d - a/2)
, _ 0.003
"
193.92 306.08
, = 0.0019 < 0.002
Use 0 = 0.65
= 0 T (d - a/2)

P - bd
1072 :0.029
p = (500)
280
7 $ Pb
=
Pimx
(0.026) = 0.0195
0.75
=
PnB
)
reinforced
(
over
>
P Pmx
'

'

Steel does not yield:


0.003 . ,
c

n Steel

sssd

bloC
ssion

0.003

o#

a:
f

5 . Problem :

sile force carried Pi

Tensile force carried by the steel bars.


600
(
d
c
)
.
f
c
600
(
500
)
306.08
.
f
306 08
f = 380.14 MPa < 414 MPa

A beam has a width of 300 mm and an


effective depth of 500 mm. fc' = 28 MPa .,
fy = 414 MPa. Es = 200,000 MPa .

Determine the depth of compression block


for a balanced condition.
Determine the balanced steel area
required.
Determine the moment capacity for
maximum steel area requirements for a
balanced condition.

Solution :
Depth of compression block for a
balanced condition.
300
;

-'

iM

Tensile force:

T = As fs
T = 4072(380.14)
T = 1547930 N
U 15 7.93 kN

Balanced Condition of
Beams Reinforced for
Tension

00
4072(6i
0.85(25)(0.85)c(280) =
c
2
5057.50 c = 122160000 - 2443200c
2
c + 483.08c - 241542.3 = 0
c = 306.08 mm
a = IJc
a = 0.85(306.08)
a = 260.17 mm (depth of compression block)

C=T
0.85 f ab = A f

con?

260 171

= 372,2 kN.m

f, = sE
_ 0.003(d - c)(200,000)
f
c
_
600 (d - c)
f,
c

as a width of 2S3
) of 500 mm. IthL
.
1072 mm2 atthebcq|, = 0.026. fe' - Z
jepth of

Mu = 0.65 (1547930) (500 -

d- c
0.003(d - c)
c

ot Yield

235

0.003

0.85 fc

295.86

500

Aab

?. v. 14

Ml
Et

ab y

mr

Si

REINFORCED CONCRETE DES

236

5 - A Problem:

600 d
f

900

A rectangular beam having a width of 30


0
effectivwe depth of -45
and an w
dliu
ryy
0 mm. ^
reinforced with 4- 36 mm in diameter
fc = 30 MPa fy = 270 MPa .
Es = 200000 MPa.
01

600(600)
C~
414 + 600
C 295.86

. .
WM

Compute the balanced steel ratio of fe

aa (ic

beam.
Compute the nominal moment capacity i
the beam.
If the value of fc' is reduced by 50X,
compute the percentage of the reduced
nominal moment capacity of the beam.

a * 0.85(295.86)
a - 251.48 mm

UI Balanced steel area required.


'

0.85

0.85

3 si

Mn =
Mn =
Mn =

Mn =

C=T

Solution:

0.85 fc ab = Asb fy
0.85(28)(300)(251.48) = Asb (414)

Balanced steel ratio.

Asi ~ 4337 mm2

p Perce
morm

0.85 fc' R (600)


pb " fy (600 + fy)

3) Moment capacity for maximum steel


area requirements for a balanced

condition.

A = 0.75 Asti

0.85(30 )(0.85 )(600 )


270(600 + 270)

pb

Pb

= 0.0544

T
p=

P = 0j

(max . steel area for balanced condition)

p = 0.

Nominal moment capacity.

A, = 0.75(4337)
2

At

= 3253 mm

= , = 0.002 < 0.005

0- 85/c

Use 0 = 0.65

Mu

= 0 A, f (d - a/2)

0,

(
)
41
32
0.
=
(
4
65
53
) (500 -
Mu

Mu

Pb * S;

= 327.62 kN.m
C=T

-ORCED CONCRETE DESIGN

J*
1

fy
As
'
=
ab
fc
0.85

<

>lP

0.85(30)(a)( 300) =

fl

Pmax " 0.75 Pb

(36) 2(4)(270)

is

As
P = bd

( |)

= Asfy d -

Anient

reduced J

Mn =

rrtage of

.70
143
(36) 2(4)(270) 450 - 2

Mn = 415.7 x 106 N.mm

oadty of ihetsy j

pmax = 0.75(0.0277)

Pmax = 0.0208

a 143.70

v.

236-A

f
p=

(36)2(4)

300(450)

Mn = 415.7 kN.m.

Percentage of the reduced nominal


moment capacity of the beam.

Since the steel will not yield, locate the


neutral axis from the top of the beam.

As
P=M
bd

f
P=

'

0)

0.003

(36)2(4)

300(450)

p = 0.03016 < 0.75(0.0554)

450 - c

s =/y /Es

p = 0.04155 (under reinforced)

= 270/ 200000
= 0.00135

larity -

0.85/r

'

#1
^

steel does not

p = 0.03016 > 0.0208


yield

0.85 fc' (J 600


Pb fy(600 + fy)
0.85(15)(0.85)(600 )
270(600 + 270)
Pb = 0.0277

0.003 0.00135
= 450 - c
c

1.35 - 0.003c = 0.00135c


0.00435c = 1.35

c = 310.34
a = lie

:i

4!

REINFORCED

236-B

C=T
0.85 fc ' ab = A, fy

MPa
5
30
'
fc
for
Note: 6 = 0.85
3 = 0.85(310.34)
a = 263.79 mm.

0.85(20.7)0(400) = - (28f (4 (4

( |)

Zi
2
)
|
450
(
)
270
)(
(36) (4

Mn = Asfy d -

Mn =

CONCRETE

Mn = 349.7 x 106 N.mm


Mn = 349.7 kN.m

a = 145.09 mm
a = Bc
145.09 = 0.85 c

c = 170.69 mm
0.003

Percentage of the reduced nominal


moment capacity:
349 7
x 100
Percentage =

170.69

Percentage = 84.1%
429.31

&
.s

5-6 Problem :

ft

*
m

*ft*

A reinforced concrete beam has a width of


400 mm and an effective depth of 600 mm. It
is reinforced for tension with 4 - 28 mm 0
bars. fc = 20.7 MPa, fy = 414.6 MPa.

Determine the percent increase in nominal


moment if the depth is increased to
700 mm.
Determine the percent increase in nominal
moment if fc is increased to 27.6 MPa.
Determine the percent increase in nominal
moment if the steel is change to
4 - 32 mmo.

0.003
429.31 170.69
, = 0.00754

e"

414.6

200,000
y = 0.00207

> y (steel yields)

Solution:
Percent increase in nominal moment if the
depth is increased to 700 mm.
0003

0.85fc

Nominal moment if d = 600 mm

( |)

M = A, ft d -

600

d -a!2

- 4-28 mm*
400

M = 484755959 N.mm
M =484756 kN.m

Es

|
4.6
I
41
M = 0.90( 7i:/4)(28) (4K
2

T =A sfy

/
.

>

%
EggCONCRETE DESIGN
m
0
m
70
d
if
=
t
en
om
m
Honing

Percent increase in nominal moment if the


steel is change to 4 - 32 mm 0.

)
2
/
a
d
(
fy
,
eA
Mr *

236-C

)
0
)
6
(
)
(
70
4.
4
41
28
(
)
(
7
rM
90
0.
M =
m
m
.
N
3
66
60
66
57
=

Ma

C=T

0.85 fc ' ab = A, fy

0.85(20.7)(a)(400) =

= 576661 kN.m

(32) (4X414.6)

a = 189.51 mm

:
t
al
en
in
m
om
no
m
in
e
as
re
inc
Percent

(576661 - 484756)
0
10
Se
=
3
re
inC
%
484756
% increase = f 8.96%

Mn

= 0 AS

fy (d -

|)

( )(32) (4X414.6) (600 -

M= 0.90

M= 606490 kN.m
@

if
t
en
om
fc
m
al
in
m
no
in
se
ea
ina
Percent
is increased to 27.6 MPa.

% inaease in nominal moment:


'

% increase =

0.85 fc * ab = As fy

% inaease = 25 f %

(28) (4)(414.6)

a = 108.82 mm

( f)
?

)
= 0.90(I)(28)(4)(414.6) (600 -

M = A, C d

Mn = 501423048.1 N.mm
M= 501423 kN.m

C=T

0.85(27.6)(a)(400) =

li? :

(606490 - 484756)

Percentage inaease in nominal moment:


)
56
47
48
(
23
14
50
,
W cf 484756
% increase = 3.44%

<00

I
^^

REINFORCED CONCRETE DES

236- D
5-C Problem

A rectangular beam has a width of 250 mm


and an effective depth of 575 mm. The beam
is reinforced for tension to cause the strain in
the tension steel to be 0.005 just as the
maximum strain in the concrete reaches
0.003.

Amount of steel needed.

:
ic

C=T

0.85 fc ab = As fy
0.85{20.7)(183.28)(250) = As (414.6)
2
mm
1945
As =

40

fc = 20.7 MPa
f = 414.6 MPa

Determine the depth of compression


block.
Determine the amount of steel needed.
Determine the ultimate strength of the

Mu

beam.

Ultimate strength of the beam

( |)

= 0 Asfy d-

ASvJ "

C= T
0.85 f/

Mu = 0.90(1945)(414.6) (575 Mu = 350.8 kN.m

Solution:
i , . Depth of compression block

B
-

0.003

0.85fc

.,

d -af 2

As
T =A
b =250 mm

fy

: a = 104

5-D Problem

He

A rectangular beam has a width of 250


and a total depth of 640 mm. The beam
reinforced with 3 - 25 mm 0 placed with Bp 122
steel covering of 65 mm from the bottom of tl^ B
beam. fc = 27.6 MPa. Assume A 615 gra E
60 steel with fy = 414.7 MPa.

By ratio and proportion


0.003
0.005
~
c
575 - c
1.725 = 0.008 c
c = 215.63 mm

3 = 0.85 (215.63)

a = 183.28 mm

Determine the distance from


compression fiber to the neutral axis of

beam.

Determine the nominal moment of

beam.
ucam

a=8c

A, = 14

1.85(27

r
*

Determine the percentage increase jo jjl


nominal strength of the beam if|
increase the steel reinforcement
100%

l03

{REINFORCED CONCRETE DESIGN

Solution :
n
e
ssio
pre
rem
ext
com
from
fiber
ce
tan
Q Dis
to the neutral axis of the beam.
0.003

Nominal moment of the beam.

M = 1472.6(414.6) 575

M u = 319.4 kN.m
-

d an

6C

offr beani.

Es

T=

250 mm

sfy

Percentage increase in the nominal


strength of the beam if we increase the
steel reinforcement by 100%,
0.003

Assume steel will yield:

K414.6) (575 -H
;

71

5)

0.85 fc '

3- 25 mm 9

)
f

M = A, l, d -

236- E

0 851V

C=T
0.85 fc' ab = As f,

575
d

A = j (25) (3)
4
2

As = 1472.6 mm

0.85(27.6)(a)(250) = 1472.6(414.7)
a = 104.12 mm
&
1

of Si

250

a = fic
104.12 = 0.85 c
c = 12150 mm (distance from extreme
compression fiber to neutral axis)

tB

Checkifsteel wiflyield:

0.003

nof

l^
c

so

^
|

of V HI
^
^ *
<

'

.
/

=
0
1
11
0
>e =
*
-

es

S< lyields

290,000

C=T
0.85 fcab = A, fy
0.85(27.6)(a)(250) = 2945.2(414.7)

a = 208.25 mm

M u = A s f y (d -

)
1

r =As / y

M= 2945.2(414.6) 575

122.50

!e

nC
3
dlSt

E.S

= 0.0021

m
kN
.
.11
575
=
Mu

% increase = 80%

41 / 2

mm
TE
RE
NC
CO
DESI
REINFORCED

236-F

a
has
m
bea
te
cre
A rectangular reinforced con
of
th
dep
e
ctiv
effe
width of 400 mm with an
0
mm
28
4
h
wit
600 mm. It is reinforced
,
a
MP

7
20.
=
fc
bars at the bottom
fy = 414.6 MPa.
,

in
se
rea
inc
e
tag
cen
per
the
Determine
ent
cem
for
rein
el
ste
the
if
nt
me
mo
l
ina
nom
is changed to 4 - 32 mm a .
Determine the percentage increase in
nominal moment if the effective depth is
increased to 700 mm.
Determine the percentage increase in
nominal moment if fc is increased to
27.6 MPa.

A,

(d - f )

* 46*6
;

i
.

for
k
ec
Ch
m
Mu = ;

(32) (4)

A, = 3217 mm

| 40(!
HI

8
Mu = 405

C=T
0.85 fc' ab = A, fy

Irate rr

ftebeai

a = 189.51 mm

9 Cut-of

Percentage increase in nominal moment if


the steel reinforcement is changed to
4 - 32 mm a.
0.003

0.85fc

Mn

'

- :

= 3217(414.6)

(eoo -

M = 673.88 kN.m

/ a

3-28 mm

suppo
2

PMu = 466

0.85 (20.7) a (400) = 3217(414.6)

Solution:

T
P
0.9

M= 538.62 kN.m

jit ' '


5#

(
)
.6
414
600
(
3
246
=
145.09)
Mn

When A = -

9r

,
*
23

Percentage increase in M.
T -Asfy

40

/0 increase =

= ~ (28) (4)
2

A, = 2463 mm

C=T
0.85 fc ' ab = A, fy
0.85(20.7){aK400) = 2463(414.6)
a = 145.09 mm

53862
^^

<

% inaease = 25.11%

&

2)

38.!

0;

i
H8B1
PIETE DESIGN
a
2

237

mom
ent capacity of the beam:
Ultim
ate
2
Mu = 0l(d - )

( )

6) 600.

<

145.09)

' .m

~ )
d
(
Asfy
0.90
Mu =

( )

Mu = 0.90 j ( 25) (4)(414.7) 675


Mu = 466.4 x 10 N.mm
Mu = 466.4 kN.m
Check for actual moment:
2
WuL
Mu =
8
2
40
9
(
)
Mu :
8
Mu = 405 kN.m < 466.4 kN.m (safe)

(32)J (4)
2

fr

I00) = 3217(414.6)

Wu 40 kN/m

H)

*~ X

180

180

Mu = 180(x) 240.3 = 180x - 20x 2


2
x - 9x + 12.02 = 0
9 5.74
X=
2
x = 1.63 m. (cut off point of 2 bars)

Alternate Solution:

' =40 kN/m

Cut off point of two of the bars from the


support:

)
2

' 189
)
6) (eoo"T
^

O;

lIHii ate moment capacity of


Ultim
the beam is Mu = 466.4 kN.m

450

L |

2 -25 mm 0

.j J

9m

180

180
i

0.85 fc '

0.003
iv

-c

=40 kN/m 2-25 mm 0

c =38.57

dJl2/f T
/ rf-c=655.72
~

2 -25 mm 0

T =Asfy

l.m

2- 25 mm a

9m

Ej

Remaining steel bars is 2 - 25 mm a

ease

:
inM
.

,
53862 00

573

538-62

,11%

As =
)

164.7

2
(
j 25) (2)

= 981.75 mm2

C= T
0-85 fc ab = As fy
085
-85(27.6)(a)(450) = 981.75(414.7)
a = 38.57

Mu : 0 As fy (d - a/2)

Mu = 0.90(981 75)(

</

414 7) 675 -

= 240.3 x 106 N.mm


Mu = 240.3 kN .m

240.3

'

4.5
4.5
Using squared property of parabola:
1647
405
2 "
2
(4.5)
a
a = 2.87 m. from center

x = 4.5 - 2.87
x = 1.63 m. (from support)

*
'

IfJkuf

CONCRETE
REINFORCED
DEs|g

238

Determination Whether

1963.5
P - 300 (380)

Tension Bars is needed

p - 0.017
pma - 0.75 pb = 0.023
p < pmax
Only tension bars is needed.

Compression Bars or

Q|Q3s SZZESE12u3l

A 12 m. simply supported beam is provided by


an additional support at mid-span. The beam
has a width of b = 300 mm and a total depth
h = 450 mm. It is reinforced with 4 - 25 mm 0
at the tension side and 2 - 25 mm 0 at the
compression side with 70 mm cover to
centroid of reinforcements. Fc = 30 MPa,
fy = 415 MPa. Use 0.75 pt, = 0.023.

Therefore the beam needs only


bars as specified in the problem.
C=T
0.85 fc' ab = As fy
0.85 (30)(a)(300) = (^/4) (25)2 (4X415)
a = 106.52 mm
(depth of rectangular stress block)

Determine the depth of the rectangular


stress block.
Determine the nominal bending moment,
Mn.
Determine the total factored uniform load
including the beam weight considering
moment capacity reduction of 0.90.

Nominal bending moment:

Mn = Asfy d -

Mn = - (25)J (4)(415) 380 Mn = 266.2 x 106 N.mm


Mn : 266.2 kN.m

Solution:
Depth of the rectangular stress
block:
0003

0.85fe '

310 -

|s

Check if compression bars is


needed.
As
P a TT
bd

A, =

f1963.5

unmniHiiuuH
S

50

A> =

Total factored uniform load includinj


beam weight:
c

</=380

?
y

(25)2 (4

mm2

R
T

'

I.

8=1 .
384 El

^
.

L
48

"

El

fv.
Po

0 |

ft

GN
SI
DE
E
ET
R
C
N
O
C
D
E
REINFORC
5 wL " PLa
3M 48 El
SwL
P=
8
2R + P = wl
5 wL
,
2R = wL g
3 wL
OD
2Rl

239

0 023

' ' '****


S 5

bsam

_
r
3

rf n the proi

'

t
)
(
)
(
)
M, = R (| - W |

MB

32

uniform
102

200000
4700 >/ fc'
200000
4700 V242
n = 8.65 say 9

^
.

239.58 =

A rectangular reinforced concrete beam has a


450
of
th
dep
width of 300 mm and an effective
mm It is reinforced for. tension at the bottom
with a total steel area of 962.5 mm'v fc = 24.2
MPa fy = 345 6 MPa . Es = 200000 MPa .

Evaluate the curvature vfy due

g
ldin
yie
ial
init
es
duc
pro
moment My which
f the tension steel in radians per m.

450-x

er=?

'

CURVATURE of
BEAMS
r

Es
n"
Ec

32
2
w(12)
239.58
32
w = 53.24 kN 1 m

.m

Curvature y/y due to moment My which


produces initial yielding of tension steel:

MB = Mu = 0.90 Mn
)
.2
266
(
0
0.9
=
Mu
.m
kN
.58
239
=
Mu

315.10

Solution:

wL

(4)(415) 380

3 wL wL
8
16(2)
2
2
3 wL - 4 wL
2

ing moment:

0* N mm

45=962.5 mm2

angular stress

AL

450

300

R = wL

ify
0) = (R/4) (25)i

Evaluate the curvature % due to nominal


flexural strength Mn of the cross section in
radians per m.
Determine the magnitude of the cross
sections curvature ductility ratio

M
| = n/1s (450 - x)

150x2 = 9(962.5)(450 - x)
2
150x + 8662.5X - 3898125 = 0
2
x - 57.75x - 25987.5 = 0
x = 134.90 mm
450 - x = 315.10 mm

345.6
y
200000
ey = 0.001728
"

REINFORCED

40

BEAM REINFORCED

e
Curvature y/y = 45Q - x

FOR

Given data :
Design moment
Mu
b
Width of beam
Effective depth of beam . . . d
f'
Concrete strength
Yield strength of steel . . . . f

Curvature y/u due to nominal flexural


strength Mn:

0-003
=
*

450

mm

COMPRESSION

reinforcements.

0.00548 rad /m.

P' \

pbd

Required :
a) Moment to be carried by
compression bars
M2
b) Total steel area in tension . . As
c) Steel area in compression . . As '

0- 8S/C

JSmrai

As =9

T=Asf

I
R

0.85fc '
d'

Asfy = 0.85 fc'ab

idn\

962.5(345.6) = 0.85(24.2)(a)(300)
a = 53.90
a = Rc

0.003
Vu 63.42

if /u = 0.0000473 rad/mm
y/u = 0.0473 rad/m

%
Curvature ductility ratio =
Wy

* fMk

CO

CDR - 8.63

T=C

53.90 = 0.85c
c = 63.42 mm
0.003
W/ =

o.oo

A. Design for dimension and steel

0.001728
Vy ~ 315.10
y/y = 0.00000548 radians/mm
\ffy =

&

Cj=4s7s '

(d -d )

Tr* sif, TrzAsi!>

Moment carried by the compressionk


To obtain a reduction factor of 0.90, the
max. reinforcement ratio corresponding
to a net tensile strain of 0.005 is equal to:
0.003
003
0
/
V
j
P = 0.85 ft
f ' 0.003 + 0.005'
a)

Assun

A =/
C=T
.85 f

a=

=0

,=M
b)
Tots

a=G

Ca

'

Frorr
died

From the strain diagram:

P- 7

0.003

: d

1*

Et =0.005 0.003

BSpRCEDCONCRETE DESIGN
d
0.003 0.003 + 0.005
0.003
=
d 0.003 + 0.005
'
0.85
=
fy
fc a b
As

241

"

Sfon

A#

beam .

V
d

;f

teel . . f
i

riedby
rs

III

^
pbdfy

p
H=

-\

&

r =^i/ A

M2 = 0 As2 fy (d - d )
M2
.A
2
,
=
82
0 fy (d - d )
A, = A + Aa
1

0.85 fc B (0.003)
fy (0.003 + 0.005)

0.85fc

( d -aP

0.85 f ' ab = A

liM
+ )
fy ' 0.003 0.005'

c) Steel area in compression :


If compression bars will yield
AS2 = As'

, fy

_L_

^
, ,

a= -

fy

.^

(d - a/2)

M2 = MU - M,

tgram

005

._

40.

bj Tota/ sfee/ area in tension :


a = lic
a
c= li
From the strain diagram,
check whether compression bars will yield
g'
0.003
c - d'
c
0.003 (c - d )
e
*
c
'

If compression bars will not yield


As f vr
600 (c - d')
where : f ' =

0.85 f ' b

M =0A

AS2 = a

A '=

&

E
Compression bars will not yield
600 (c - d )
Use : r
c

= 0.85 f li c b

As1 = pbd
C=T

2) When e;<

bd

Assumed : p = 0.85

'

0.85 T lie

Compression bars will yield


(V = f )

fy = 0.85 fc' R c b

itension . . A
ipression . . A
Cf
Ci +

;i
E

1) When e >

ii

^V

B. Compression Bars Will Yield


Given data :

Total steel area in tension . . At


Steel area in compression . . At
: b
Width of beam
Effective depth of beam . . . d
f'
Concrete strength
Yield strength of steel . . . . f
L
Span of beam

'

'

CO
REINFORCED

242
Required :
a) Depth of compression block
b) Ultima!moment capacity
c) Safe uniform liveload

a
M

rf

0.85 fr

1.7
L
D
+
1.4
=
wu

t-t

it axiki carry
**

c) Safe uniform liveload


}
W, l
M

C. Compression Bars Will Not Yield

, gJ>

rrfT
(rf

{ d -d )

/2

,=ASIf

Ti=Asif

a) Depth of compression block :


Check if compression bars is really needed.
K

bd
P , - 0.75 p
If P > P (compression.bars are needed)

repression bars will yield if the

Given data :
Total steel area in tension . . Af
Steel area in compression . . At '
'
b
Width of beam
Effective depth of beam . . . d
f'
Concrete strength
Yield strength of steel . . . . f
Span of beam
L
Required:
a) Depth of compression block . . a
b) Ultimate moment capacity . . . Mu
c ) Safe uniform liveload
it could carry
LL

blowing condition occurs:


0.85
'
'
Rd
600
fc
P ^
fy d (600 - fy )

'

bd
f,' = fy (steel in compression yields)
T = C + C2
As fy = 0.85 fc ab + As fy
Solving for a:

0.85 fc ' b
b) Ultimate moment capacity :

a = Bc

j
)
+
(
d
d
'
CJ
| -

M= C1 (d -

C = 0.85 fc ' a b
C2 = As fy
Nominal moment capacity:

- ) + A,' fy (d - d')
Mn = 0.85 fc' ab (d |
Ultimate moment capacity:
Mu = oMn

0.85fc '

(<d -a /

(i 4

21

i 8

ft

r.T=V>

a) Depth of compression block :


Check if compression bars is really needed

h,

pOfe

bd
Pnax = 0.75 pb

,fP

)
needed
(compression bars are

>P

Compression bars will not yield if the


following condition occurs:
n .r > * M5 fc' Rd' 600
fy d (6d0 - g
.

fs

fy (steel in compression does not

UseV =Hii I

NCRETE DESIGN

,
,

T = C C?
A, f = 0.85 f a b + A,' f,'
{

Solving fa a:
A, f A;V

243
b

*1

0.85 fc

Cl

C2~AS

(d-d )

(d-a / 2 )

&
-4 i
-

0.85 fc ' b

TrASlfy

fy

&
,

TflAS2f

b) Ultimate moment capacity :

a = Sc

(d -|) * C2 (d - f)
(

C, = 0.85 fc a b
C2 = A,' V
Nominal moment capacity:

Vj

Mn = 0.85 fe' ab (d -

(
)
f
'
)
d
+
A
T
,
<
/
|

fy

Ultimate moment capacity:

Mu = 0 Mn
. . . LL

c) Safe uniform iiveload :


?
WL
8
Wu = 1.40 + 1.7L

fc

l -all )

*'
SI

D. Balanced Condition for Beams


Reinforced for Compression and
Tension:

>

Given data :
Total steel area in tension . . A,
Steel area in compression . . A,'
b
Width of beam
Effective depth of beam . . . d
f'
Conaete strength
Yield strength of steel . . . fy
L
Span of bean

s wm

v "

'

Required :
a) Depth of compression block . a
b) Balanced Steel area
c) Maximum Amount of steel

area permitted:

a) Depth of compression block :


Balanced Steel Ratio,
(1) If compression bar: will yield:
- 0.85 fc' R (600)
P
Pb
fy (600 + fy )

(2) If compression bars will not yield:


~_ 0.85 fc' R (600) +t , V
P
Pb
(600 + fy )
fy
600 d
c=
600 + fy
0.05 (fc' - 30)
B = 0.85 7
(but not less than 0.65)
8 = 0.85 for fc < 30 MPa
a = IJc

b) Balanced Steel Area :


C + C2 = T

0.85 fc ' ab + As ' fy = Asfy


(A, - A,') fy = 0.85 fc' ab

AS - A ' = AS1

_ 0.85 fc' ab-

r
AS

Atf = '
Ast = Asi + AS2
c) Maximum Amount of steel area permitted:
Max. As = 0.75 ft bd + As'
_ 0.85 fc' R (600)
fy (600 + fy )

243-A

CONCRETE
REINFORCED
DESl
Solution:

of
width
a
has
beam
supported
A simply
It
.
mm
330
of
depth
300 mm and an effective
is reinforced at the bottom with a steel
2. Assuming
mm
reinforcement area of 2464
steel covering to the centroid of the
reinforcement is 70 mm. fc' = 34 MPa,
fy = 415 MPa.

Determine the ratio of the depth of


compression block to the distance of the
neutral axis from the top of the beam.
Determine the balanced steel ratio.
Determine the depth of compression
block.
Determine the total compressive force
carried by concrete.

Determine the value of the moment


reduction factor 0.
Determine the design strength of the
beam.

TOP

Determine the distance of the neutral axis


from the top of the beam if it is reinforced
with a steel area pf 1232 mm2, at the top
in addition to the existing steel area of
2464 mm2 at the bottom.
Determine the stress of the compression
bar.

'

0.003

f7
T

'

11

N A.

At .

Value

- = ft
c

.
^
0.85

0.05(34 - 30)
8 = 0.85 7
II = 0.82

a=B<
117.94
c = 14

Balanced steel ratio:


0.85 feII 600
Pb
fy (600 + fy )

_
_
0.85(34)(0.82)(600)
Pb

186.1]
le, = o

415(600 + 415)
Pb = 0.034

B = 0.(

0 = 0.1

|8 = 0

Depth of compression block:


300

Determine the design strength of the


beam using moment reduction factor 0 =
0.90.
Determine the concentrated live load it
could carry at the mid-span in addition to a
'of 20 kN/m
load
dead
including its own
weight for a span of 6 m.

,J

Ratio of depth of compression block t


distance of the neutral axis from the tar
the beam.

0.85/c

3
</=330

70

*B
143.83

, *

\
h
*
1 JJ ,
, f,
l

T* A

%0
esigr

k,

/
/1

yt =>246 ran?
'

0.003

C=T
0.85 fc' a b = As fy
0.85(34)(a)(300) = 2464(415)
a = 117.94 (depth of compres

i
^
^

f:.

SI
J%3

243-B

TE
RE
N
NC
SIG
DE
CO
ED
RC
0
1NF
by
rete
:
conc
ied
carr
e
forc
sive
pres
com
|*
C s 0.85 tc a b
| C = 0.85(34 X117.94)(300)

the
of
top
the
from
axis
ral
neut
Distance of
at
ed
plac
are
bars
l
stee
beam if additional
the top:

C = 1022540 N
C * 1022.54 kN

If

300

0.003

0.85/f

ip

Value of moment reduction factor 0:

330

LfTS-rJli-: PLA

300-C

A *2464 nun?

0.003

C2

70

Tl

C=143.83

330 C=186.17

Check whether compression bars will yield:

a = Bc
117.94 = 0.82 c
c = 143.83
0.003
,
'
186.17 143.83
, = 0.0039 < 0.005

Compression bars will not yield.

,-

0 = 0.65 + (0.0039

- 0.002)

0 = 0.81

til
My

,AM

* M. = BA f, (d -

Design strength of the beam:

)
|

K = 0.81(2464)(415) (330 = 224.5 x 10* N.mm


- 224.5 kN.m

5'

, 0.85 fc' B d' 600

when p p <

(250)
0 = O.65 + ( 0.002)
3

f d (600 - fy )

i!

Compression bars will yield.

, 0.85 fcBd' 600

-m.

A
P= b d
2464
P = 300(330)

p = 0.025

TL

m
%

REINFORCED CONCRETE DESIG

243-C

too

1232
300(330)

p' = 0.0124

= 0.0126

V:

0.85(34)(0.82)(70)(600)
415(330)(600 - 415)

0.85 f/ l3 d' 600


0.0393
=
fyd (6O0 - g
0.0126 < 0.0393

if

;
>d

-an.)

0.003(c 70)(200,000)
C

600 (c - 70)
G

C, + C2 T, + T2

C1 + C 2 = T

0.85 fe' a b + A,' f,' = As fy


0.85( 34)(a)(300) + 1232 f,' = 2464(415)

NA

__

c,

70

0.85 fc (3 d' 600


f d (600 - f )

, 246* nun'

p - p* = 0.025 - 0.0124
p - p'

rf 330

0 85/r '
,

__

If

bd

8670a + 1232 f,' = 1022560


Steel in compression does not yield:

a = (3 c

V*

a = 0.82 c

fy
0.003
70

c
C

IH-

-70

I*

8670(0.82) c +

1232( 60

HC :3 = 10

c - 39.86c - 7278.25 = 0
2

c = 107.54

g;

Stress of compression bar

c 70
0.003 (c - 70)

600 (c - 70)

0.003

E, '

V
5

IJ

c
600 (107.54 - 70)
107.54

Vs 209.45 MPa < 415 MPa

243- D

RCED CON

c '

NA

Ml

J
71

'

on
ing
cti
us
du
r
gth
re
en
cto
fa
str
n
sig
De

0 = 0.90

.-
300

70

0.85/c
C2

A ,' 2464 ram

rf =330

(d an )

( 200,000)

70

MU = 0 M
M= 0.90(285.68)

=2464 mro?

(d d' )

T2

Tl

kNjn
1
7.1
25
=
Mu

Live load it could carry.

a = Bc

1
nmiinmiiiHi

3 = 0.82 (107.54)

wDL =20 kNVm

a = 88.18

: =\ \

6m

+ 1232 f,' = 2

= 1022560

C = 0.85 fc ' a b

, = 0.85(34)(88.18)(300)

6000

& w m L v .4)
'
_
P(6)(1.7) , 20(36)(1.4)
M

C = 764521 N

C2 = A;V

232(

Mu = 51.42 kN

C2 = 1232(209.45)

C2 = 258042

8.25

'

Mn = C, (d -

C
+
)
j

(d - d )
1

M= 764521 330 - M

SSi

:
r
3
nb

+ 258042(330 - 70)
s

M= 285.68 x 10 N.mm
M= 285.68 kN.m

P
REINFORCED C

244
Stir I Areas Are

0,

&

Required

Problem :

A rectangular beam that must carry a service


live load ot 36 kN / m and a calculated dead
load ol 16.4 kN /m on a 5,4 m , simple span is
limited In cross section for architectural
reasons to 250 mm and 50 mm total depth.
Assume that the tension steel centroid will be
100 mm above the bottom face of the beam
and that compression steel if required will be
placed 82,5 mm below the beams top surface,
f,/ * 27,6 MPa ,
f 414.7 MPa .
<D Determine the remaining moment to be
carried by the compression steel.
Determine the steel area In compression .
( )
3 Determine the steel area in tension

Solution:
CD Remaining moment to be carried by the
compression steel .

::
l

B
Wg ' 1.4 DL + 1.7 Li
W " 30(1, 7) + 1.4(15.4)
W * 72.56 kN/m

wm

400

dan

<5,
r/ " Vr

<&

,
dd

WUL
8

M * 264.48 kN. m

To obtain a reduction factor of 0,90, the max.


reinforcement ratio corresponding
to a net tensile strain of 0.005 is equal to:

A,, pbd
A = 0.018(250)(400)
2
A = 1800 mm

[L J

pressw
Co^
'
E
fs
V=
f = 0.00175

!=

C=T
0.85 fc' a b = A fy
0.85(27.6)(a)(250) = 1800(414.7)
a = 127.27 mm

350 MP

f
Atf
=
:
106
x
37.48

= fy (d - )
M , = 0.90(1800)(414.7) (4OO - ~
M , = 226 kN.m
M,

0A

Mj

306 mi
=
*

m i -fo

A, , = As' f
306(414.7) =
\' = 363 mn

0,003
'
250.27 149.73
C, = 0.005

_
e;

0.003
"
87.23 149.73

tegular be

87

250.27

nsio
*
at
N
"

i mPression

Aii

et = 0.00207

>

i
tf

e, = 0.00175 < ey = ~
414.7
200 000

As = Asi + As
As = 1800 + :
As = 2108 mi

Steel area ir

Steel area in compression .


a = Rc
0.003
127.27 0.85 c
c 149.73
149.7:
C

p 0.018

M? MU - M ,
Mj = 264.48 - 226
M 2 = 38.48 kN .m

'

250

a,

Assume value of p :
0.85(0.85)(27.6)(0.003)
P^ ~
414.7(0.008)

th,

Pan

'

"

BH&SRCEP CONCRETE DESIGN

Value of p .

,
84

ipS

47fo35

250

- *

0 S5 fc

bd
18< 250)(400)
300 mm*'

Depth of compression block .

c?

400

Solution:

S'.

<5.

-a n

Tt - ^Sjfy

\M

dd

300

rW 5!]
T2 = AS fj
2

A, = 4825 mm

P bd

MSVV

4825
P
300(600)
p = 0.0268

306(414.7) = AS' (350)


:
'
mm
363
A, =

ss
pre
com
in
rea

'

0.003
=
14073
_ 0.003

00207

|A rectangular beam has a

"

Determine the depth of compression


block.
Determine the design strength using a
reduction factor of 0.90.
Determine the concentrated live load at
the mid- span in addition to a dead load of
20 kN/m including its own weight if it has a
span of 6 m.

0.003

V7a

hv Y /

600

- f i x'

:-i - 4

005

!00 000

'

300

62

width of 300 mm
and an effective depth of 600 mm. The steel
2 and
mm
4762
is
area of tension at the bottom
49
that of the compression side at the top is
987.5 mm2. Steel covering is 62.50 mm from
the compression face of the beam.
A = 0.034, fc' = 34.6 MPa, = 414.6 MPa

= 0.85 c
1.73

14973
<
1.00175
4147 _

' "a

= 0.75 p
P
6 600
0.85
V
Pt
f, (600 + f )
0.05 (V - 30)
6 = 0.85 7
8 = 0.817

Steel area in tension


As = As1 + AS2
As = 1800 + 306
As = 2 f 06 mm2

Compression Bars will yield


0

64.48 - 226
8.48 kN.m

A, = 7 (32) (6)

, -

fnrfi

1A

M, = oAs, f (d tf)
37.48 x 10* = 0.90 A , (414.7){ 400 - 62.5)
*
1
A = 306 mm
^
T;= C

26 kN.m

III

l 2t m

el

0 003

600

Compression steel does not yield


V = sE,
= 0.00175 (200 000)
,f = 350 MPa

ab =M
6)(a)( 250) = 1800(4147:
.27 mm

90( 1800)(414.7) 400 -

245

P>
pt

P
P

\
e

0.85fc

>r*t
/2 )
-uafl

(d

0.85(34 6)(0.817)(600)
414.7(600 + 414.7)
= 0.03426
= 0.75(0.03426)
= 0 025695

Compression bars is needed:

P >P
0.0268 > 0.025695

7/ >

REINFORCED CONCRETE D
s

Mu = oMu
Mu = 0.90(1031)
= 927.9 kN.m
(D

Concentrated live load at the mid-span

in addition to a dead load of 20 kN


/m
including its own weight if it has a span
of 6 m.

Pes'

9n stre 9th using a


factor of 0.90.

"

0.003

reduction

n o

d-d

lilt

bo

s#

^(

/
ab

BBJfCED CONCRETE DESIGN

c? (d

^
2J

Solution:

d- .

ab
'
W 6)(
18064K3(yf
87 N

compre
ssion
block
of
Depth
$
2

- (28) (4)

As =

'

. sv/

As - 2463 mm

As' = -- (28f (2)


As' = 1231 5 mm*

^
302-

+ 407132 (&

oK

62i;

x 106 N.mm

I
1031)
IkN.m

T = Ci Ca
A fr = 085 If ab r As V
*2463(415) = 0>85( 36)(a 300)

A..

fcd-

70)
me
12315

2463
~P 330(330
;
p = 0.0248
/>, - 075 p,
.p
= 0.75(0031)
pw = 0.023

1022145 = 7650a +

^^

for
merit
Therefore
comoression
reinforce

II
of
l to a dead load
.
is
needed
its own weight if i la
Compression bars wii not / ed if -

.
Id
055
7600
8
<
p-p <

f/ d (600 - fy)
0.85 (30)(0.85)(70/ yX3)
415 (330) (600 - 415)

4(20)(6f ,

% Design strength using 0.90 as reduction


factor:

p=

300

OtH

"

A(
'd

C/

qr

rq

Tl

_
i i'

@ ^

jorted beam
w
#
t the bottom
le

beam.

nforcement
f the

beam

p - p> ( 0.036
0.0248 - C.C124 < 0.036
0.0124 < 0,036

w>I. Therefore compression bars wfl not yield.

OOmmaod
Vl
*
.
3
*
MP
0003
415
|
=
=
C
c - 70
Jk
1
. W03(c 70)

- S^ ht ^

rr-

5'

o.m

0#
OoM
/

*
'
rthedes

fy

V=0

c c TO

....

p - p< 0.036

rkN

ited live load atttei

1.7 PL

738900(c - 70)

c
73890 c:
1022145 = 7650(0.85c)
c
157.19c = cr^ + 113.63(c - 70)
1
c* 4356c - 7954.10 = 0
43.56 183.61
c== 113.59
2
a=8c
3 = 085 (11359)
a = 96.55 (deptfi of compression block)

pmax

.
WJi

m (6oo -

^
__F'

5(414.7)
I2

L
3;

247

Z
*

7/

C. = 055 f ' ab
C, = 055(30X9655X300)
C, = 738 60750 N
C2 = A,' V
f,* = ,' Ft

..

. 600 (113.59 - 76;


113.59
f ' = 23C 25 41 3

7?

ay/ '

m
'

REINFORCED CONCRETElia

l
^

248

c,
C, = 1231.5(230.25)

Solution:
Depth of compression block fw f
balanced condition.

C = 283553

,( |
- )

250

M, = C d

M = 738607.5 330 -

M, = 208 x 10 N.mm
M 2 = C2 (d d )
M2 = 283553 (330 - 70)
Mj = 73.7 x 10 N.mm
MU = 0 (M + M2 )
Mu = 0.90(208 + 73.7)
= 253.53 kN.m

62.5 |

c
</=625 mm. _ _

253.53 =

(1 -7)

^
4

WL

8
(1.7) +

a
fc
0.85

20
(
0.85 +

:
47
ASb =

T
*

0.002
625 - c
c
0.005 c = 0.003(625)
c = 375 mm

0.003

11 (1.4)

= 0.0Z
Max. As =
Max . As :

Check:
600 d
c=
fy + 600
600(625)
c
400 + 600
c = 375 mm
a = Rc
a = 0.85 c
a = 0.85(375)
a = 318.75 mm

Balanced Condition of

Beams Reinforced for

Compression

12. Problem:
A reinforced concrete beam has a width of
250 mm and an effective depth of 625 mm. it
is reinforced for compression having a steel
area As = 1250 mm2 with a steel covering of
62.5 mm measured from the center of the
steel reinforcement to the top most fibers of
the beam, fc = 20.68 MPa, fy = 400 MPa
E = 200 000 MPa.
Determine the depth of compression block
for a balanced condition.
Determine the area of balanced steel Asb
for the given cross section.
Determine the maximum area of flexural
steel in tension permitted in the given
cross section as required by the NSCP
Specifications.

.
C
+
Ci

0.8

400
> =
200 000
ey = 0.002

permitted
:
.
Max As

E,

( I '4)

|Np

Maximurr

es

P = 50.01 kN

625-c

Concentrated live load at mid-span:

Mu

JE

0.85 fc*

'

PL

0.003

A
|rectangular
I fd has an eff

| MPa,
P
i
2

0*mm
*
E
*
f
M
fc = 25

factor.

Area of balanced steel Asb for the


cross section.
250
0.003 0-8lfI
62.5 l
312.5

css

mine

'

ne

ix "

375

< =625 mm

SSi(

,'

_ 0.003

"

312.5 375
e;= 0.0025 > y = 0.002

( steel in compression yields)

MffiPORCEP CONCRETE DESIGN

248-A

%' * fy = 400 MPa

Solution:

Ci + Cz = T
0.85 fe ab + A,' fy = A8b fy
,85(20 ,68)( 318.75)(250 )
+ 1250(400) = Asb (400)
2
4752
=
mm
ASb

'

Moment capacity if At * 6000 mm2.


T C
A, 1f = 0,5 (fab
6000(300) * Q.85(25)(a)(375)
a * 225,88 mm
a * IJc
225,88 = 0,85 c
c = 265.74 mm
e,
0,003

Maximum amount of tensile


permitted .
Max. A, = 0.75 (\ bd + A,'
.0.85 fc' ft (600)
fy (600 + fy )
0.85(20.68)(0.85)(600)
400 (600 + 400)
I ft = 0.0224

steel

184.26

0.00208
=
et

A
V

300
ty " ,
200 000
Ey = 0.0015 < 0,00208 (steel yields)
f
)
/
a
d
0
(
2
A
=
,
fy
Mu

.H

Max . As = 0.75(0.0224)(250)(625) + 1250


Max . As = 3875 mm2

,90(6000)(300) (450 0
=
Mu

Mu = 546 kN.m

12 A Problem:

A rectangular beam has a width of 375 mm


and has an effective depth of 450 mm.
V * 25 MPa, fy = 300 MPa.

if 0 Determine the moment capacity if it is


f reinforced with steel area in tension As =

Q) Depth of compression block if As'

mm2 is at the top of the beam.

4[

_2Z*_

compression ,
575

0.003

am
II

A ,*6aoo

id am

A,+*600fi

r,

I 4 26

(d

<>V 1s'
*

-a /2 )

TlASI /

bd
A'
d= P
bd

&

(d

-d )

Tl st f,

6000
0.036
375(450)
3000
= 0.0178
=
375(450)
0.85 fr * a d* 600

when 1p - Hp >

fyy d (600 - fy )

0.85(25)(0.85)(60)(600)
0.036 0.0178 >
300(450)(600 - 300)
0.0178 > 0,0161

265.74

|B

= 3000

"

Iif "

. . jiff ,

with steel covering of 60 mm below the top


of
the beam.
If
i Determine the moment capacity of the
beam reinforced for tension and
a '%

0.85/, '

450

6000 mm2. Use 0.9 as moment reduction


factor.
I Determine the depth of stress block if steel
'
2 is added
in
'
mm
f compression As = 3000

265.74

ip
y
REINFORCED CONCRETE D

248- B

Solution :

Compression bars will yield;


f ' =f

K , * V,

'
Atf =

Depth of compression block

3000 mm

000 = A+ 3000

*
a Af = 3000 mm

A
3000
=
=
M 280

bd

0.007

280(510)
0,85 ft' S (T 600

WVT,

'

0.85 T ab + A,' fy = A fy
0.85(25)a(375) + 3000(300) = 6000(300)
a = 112.94 mm

Moment capacity of the beam reinforced


for tension and compression .
Ml = Ast fy (d - a/2)
Mi = 3000(300)( 450 - 112.94 /2)
Mi = 354.18 kN.m
M2 = AS2 fy (d d )
M2 = 3000(300)(450 - 60)
M2 = 351 kN.m
Mu = 0 (Mi + M2)
Mu = 0.90(354.18 + 351)
Mu = 634.66 kN.m

12-B Problem:
A rectangular beam has a width of 280 mm
and an effective depth of 510 mm. Steel area
in tension As = 3000 mm2 and steel in
compression As = 1000 mm2. Steel covering
for steel in compression is 50 mm . fc = 35
MPa, fy = 400 MPa.
(D Determine the depth of comp
ression
block.
Determine the actual stress of
compression bar.
@ Determine the design moment capacity of
the beam. Use 0.90 as moment reduction
factor.
,
I

a,

Mg

A *

1000

510

0,85fc.'

cr

p
0
-41 /2)1

000.

so!

\d 3n

0003

(steel in compression will not yield)


0.05 (f/ 30)
B = 0.85 -

7
0.05 (35 - 30)
6 = 0.85 7

6 = 0.81

<

0 85 35)(0 8
0021 - 0007 <
400( 510X600 - 400)
0.014 < 0.0177
V f,
e; _ 0.003

^
. O.OOstc - SO)

es =

V = e ' E,
,, = 0.003(c - 50X200 000)
ft
600 (c - 50)
,, =

'

:
c

T = C + C2
A, fy = 0.85 fc' ab + Al' fs'
3000(400) = 0.85(35)(0.81) c (280)
x 1000(600)(c 50)
c
3
1200000c = 6747.3c + 600000c 2
6747.3c - 600000c - 30,000,000 = 0
2
c 88.92c - 4446.22 = 0
88
.92

16
0.2
9
c=
2
c = 124.60 mm
a=6c
3 = 0.81(124.60)
in
a = 100.93 mm (depth of compresS*

-X -

glNFORCnD CONCRETE DESIGN


on
ssi
pre
of
com
r
ba
ss
stre
.
l
tua
Ac

,, 600(c - 50)
'
=
f
m

a) Effective Width of Flange:

c
600(124.60 - 50)
V
124.60
a
MP
3
9.2
'
35
=
fs

!0
)

249

A FORT-SECTIONS:
Limits of the effective flange with
"b" as controlled by NSCP:

1@ Design moment capacity of the beam.


M2 = A;fs' (d - d')
M2 = 1000(259.23)(510 - 50)
M2 = 165.25 kN.m

"
"
b
of
lue
va
st
alle
sm
e
Us the
'

Slab

M, = C, (d - a/2)
'
M, = 0.85 fc ' ab (d - a/2)
M = 0.85(35)(100.93)(280)(510 - 100.93/2)
M, = 386.35 kN.m

l/\

Flange

Uv-

ML

Web or
stem

11X

600 - 41]

+
)
M
(
=
M
0
,
2
Mu

Spacing o( Beams

0 i

of

\1
\

of span

b = center to center spacing


of beams

B FOR L- SHAPED FLANGE:


b

A. Steel Area required with Neutral


Axis on the Flange:
C |2f

^
b = 16t + b

ANALYSIS OF
T-BEAMS

Given data :
Design moment
Thickness of flange
Width of web
Effective depth of beam . .
Concrete strength
Yield strength of steel . . .
Span of beam

b'

b=

)
5
6.3
+
5
38
5.2
16
(
0
0.9
=
Mu
m
kN
.
.44
496
=
u
M

Mu
t

bw

-MM-

d
f'

Use smallest value of "b"

\
L

Required :
a) Effective width of flange
t) Depth of compression block .
c) Steel area required

b
+
an
sP

b = J2

b' + 6t
U =
-u
b
T

As

beams

ui

center to center spacing of

'
1
i

You might also like